Convexity of $x^{2q}$ implies $mathbb{E} big[ left(X-X^{'}right)^{2q} big]leq 2^{2q-1} left( mathbb{E}X^{2q}+...











up vote
0
down vote

favorite
1












This problem is from page 25 of concentration inequalities a nonasymptotic theory of independence



$X$ is a random variable
and $X^{'}$ is independent copy of $X$



q is integer and $qgeq1$



It is said that the following inequality was implied by the convexity of $x^{2q}$



$$mathbb{E} big[ left(X-X^{'}right)^{2q} big]leq 2^{2q-1} left( mathbb{E}X^{2q}+ mathbb{E}X^{'2q} right) $$



However, I do not know how to deduce it. The first guess is using Minkowski inequality to get terms of $mathbb{E}X^{2q}$ and $mathbb{E}X^{'2q}$. The Jensen ineqaulity will give inequality of reverse direction, I am wondering how convexity is applied. Any suggestion is welcomed.










share|cite|improve this question


















  • 2




    math.stackexchange.com/questions/1102727/…
    – d.k.o.
    Nov 16 at 23:38










  • Thx. I am wondering how $$(a+b)^rleqslant a^r+b^rmbox{ if }0lt rleqslant 1 $$ is deduced from concavity of $ x^r$. I can see it throguh Bernoulli inequality but it's fine to find a unified way to do it through Jensen's inequality.
    – Rikeijin
    Nov 17 at 10:17












  • math.stackexchange.com/questions/318649/…
    – d.k.o.
    Nov 17 at 16:44















up vote
0
down vote

favorite
1












This problem is from page 25 of concentration inequalities a nonasymptotic theory of independence



$X$ is a random variable
and $X^{'}$ is independent copy of $X$



q is integer and $qgeq1$



It is said that the following inequality was implied by the convexity of $x^{2q}$



$$mathbb{E} big[ left(X-X^{'}right)^{2q} big]leq 2^{2q-1} left( mathbb{E}X^{2q}+ mathbb{E}X^{'2q} right) $$



However, I do not know how to deduce it. The first guess is using Minkowski inequality to get terms of $mathbb{E}X^{2q}$ and $mathbb{E}X^{'2q}$. The Jensen ineqaulity will give inequality of reverse direction, I am wondering how convexity is applied. Any suggestion is welcomed.










share|cite|improve this question


















  • 2




    math.stackexchange.com/questions/1102727/…
    – d.k.o.
    Nov 16 at 23:38










  • Thx. I am wondering how $$(a+b)^rleqslant a^r+b^rmbox{ if }0lt rleqslant 1 $$ is deduced from concavity of $ x^r$. I can see it throguh Bernoulli inequality but it's fine to find a unified way to do it through Jensen's inequality.
    – Rikeijin
    Nov 17 at 10:17












  • math.stackexchange.com/questions/318649/…
    – d.k.o.
    Nov 17 at 16:44













up vote
0
down vote

favorite
1









up vote
0
down vote

favorite
1






1





This problem is from page 25 of concentration inequalities a nonasymptotic theory of independence



$X$ is a random variable
and $X^{'}$ is independent copy of $X$



q is integer and $qgeq1$



It is said that the following inequality was implied by the convexity of $x^{2q}$



$$mathbb{E} big[ left(X-X^{'}right)^{2q} big]leq 2^{2q-1} left( mathbb{E}X^{2q}+ mathbb{E}X^{'2q} right) $$



However, I do not know how to deduce it. The first guess is using Minkowski inequality to get terms of $mathbb{E}X^{2q}$ and $mathbb{E}X^{'2q}$. The Jensen ineqaulity will give inequality of reverse direction, I am wondering how convexity is applied. Any suggestion is welcomed.










share|cite|improve this question













This problem is from page 25 of concentration inequalities a nonasymptotic theory of independence



$X$ is a random variable
and $X^{'}$ is independent copy of $X$



q is integer and $qgeq1$



It is said that the following inequality was implied by the convexity of $x^{2q}$



$$mathbb{E} big[ left(X-X^{'}right)^{2q} big]leq 2^{2q-1} left( mathbb{E}X^{2q}+ mathbb{E}X^{'2q} right) $$



However, I do not know how to deduce it. The first guess is using Minkowski inequality to get terms of $mathbb{E}X^{2q}$ and $mathbb{E}X^{'2q}$. The Jensen ineqaulity will give inequality of reverse direction, I am wondering how convexity is applied. Any suggestion is welcomed.







probability inequality convexity-inequality






share|cite|improve this question













share|cite|improve this question











share|cite|improve this question




share|cite|improve this question










asked Nov 16 at 23:33









Rikeijin

878




878








  • 2




    math.stackexchange.com/questions/1102727/…
    – d.k.o.
    Nov 16 at 23:38










  • Thx. I am wondering how $$(a+b)^rleqslant a^r+b^rmbox{ if }0lt rleqslant 1 $$ is deduced from concavity of $ x^r$. I can see it throguh Bernoulli inequality but it's fine to find a unified way to do it through Jensen's inequality.
    – Rikeijin
    Nov 17 at 10:17












  • math.stackexchange.com/questions/318649/…
    – d.k.o.
    Nov 17 at 16:44














  • 2




    math.stackexchange.com/questions/1102727/…
    – d.k.o.
    Nov 16 at 23:38










  • Thx. I am wondering how $$(a+b)^rleqslant a^r+b^rmbox{ if }0lt rleqslant 1 $$ is deduced from concavity of $ x^r$. I can see it throguh Bernoulli inequality but it's fine to find a unified way to do it through Jensen's inequality.
    – Rikeijin
    Nov 17 at 10:17












  • math.stackexchange.com/questions/318649/…
    – d.k.o.
    Nov 17 at 16:44








2




2




math.stackexchange.com/questions/1102727/…
– d.k.o.
Nov 16 at 23:38




math.stackexchange.com/questions/1102727/…
– d.k.o.
Nov 16 at 23:38












Thx. I am wondering how $$(a+b)^rleqslant a^r+b^rmbox{ if }0lt rleqslant 1 $$ is deduced from concavity of $ x^r$. I can see it throguh Bernoulli inequality but it's fine to find a unified way to do it through Jensen's inequality.
– Rikeijin
Nov 17 at 10:17






Thx. I am wondering how $$(a+b)^rleqslant a^r+b^rmbox{ if }0lt rleqslant 1 $$ is deduced from concavity of $ x^r$. I can see it throguh Bernoulli inequality but it's fine to find a unified way to do it through Jensen's inequality.
– Rikeijin
Nov 17 at 10:17














math.stackexchange.com/questions/318649/…
– d.k.o.
Nov 17 at 16:44




math.stackexchange.com/questions/318649/…
– d.k.o.
Nov 17 at 16:44















active

oldest

votes











Your Answer





StackExchange.ifUsing("editor", function () {
return StackExchange.using("mathjaxEditing", function () {
StackExchange.MarkdownEditor.creationCallbacks.add(function (editor, postfix) {
StackExchange.mathjaxEditing.prepareWmdForMathJax(editor, postfix, [["$", "$"], ["\\(","\\)"]]);
});
});
}, "mathjax-editing");

StackExchange.ready(function() {
var channelOptions = {
tags: "".split(" "),
id: "69"
};
initTagRenderer("".split(" "), "".split(" "), channelOptions);

StackExchange.using("externalEditor", function() {
// Have to fire editor after snippets, if snippets enabled
if (StackExchange.settings.snippets.snippetsEnabled) {
StackExchange.using("snippets", function() {
createEditor();
});
}
else {
createEditor();
}
});

function createEditor() {
StackExchange.prepareEditor({
heartbeatType: 'answer',
convertImagesToLinks: true,
noModals: true,
showLowRepImageUploadWarning: true,
reputationToPostImages: 10,
bindNavPrevention: true,
postfix: "",
imageUploader: {
brandingHtml: "Powered by u003ca class="icon-imgur-white" href="https://imgur.com/"u003eu003c/au003e",
contentPolicyHtml: "User contributions licensed under u003ca href="https://creativecommons.org/licenses/by-sa/3.0/"u003ecc by-sa 3.0 with attribution requiredu003c/au003e u003ca href="https://stackoverflow.com/legal/content-policy"u003e(content policy)u003c/au003e",
allowUrls: true
},
noCode: true, onDemand: true,
discardSelector: ".discard-answer"
,immediatelyShowMarkdownHelp:true
});


}
});














 

draft saved


draft discarded


















StackExchange.ready(
function () {
StackExchange.openid.initPostLogin('.new-post-login', 'https%3a%2f%2fmath.stackexchange.com%2fquestions%2f3001779%2fconvexity-of-x2q-implies-mathbbe-big-leftx-x-right2q-big%23new-answer', 'question_page');
}
);

Post as a guest















Required, but never shown






























active

oldest

votes













active

oldest

votes









active

oldest

votes






active

oldest

votes
















 

draft saved


draft discarded



















































 


draft saved


draft discarded














StackExchange.ready(
function () {
StackExchange.openid.initPostLogin('.new-post-login', 'https%3a%2f%2fmath.stackexchange.com%2fquestions%2f3001779%2fconvexity-of-x2q-implies-mathbbe-big-leftx-x-right2q-big%23new-answer', 'question_page');
}
);

Post as a guest















Required, but never shown





















































Required, but never shown














Required, but never shown












Required, but never shown







Required, but never shown

































Required, but never shown














Required, but never shown












Required, but never shown







Required, but never shown







Popular posts from this blog

Le Mesnil-Réaume

Ida-Boy-Ed-Garten

web3.py web3.isConnected() returns false always